Offenes Problem? Quadrat der Wellenfunktion Ψ(x)xo=δ(x−x0)Ψ(x)xo=δ(x−x0)\Psi(x)_{x_o} = \delta(x-x_0) eines lokalisierten Teilchens an einem Punkt x0x0x_0?

Kennt jemand den Stand des Problems, die Wellenfunktion (nicht-relativistische Quantenmechanik) eines an einem bestimmten Punkt lokalisierten Teilchens zu definieren?

Landau-Lifshitz sagt in Kapitel 1, dass diese Funktion ist Ψ ( X ) X Ö = δ ( X X 0 ) und gibt eine Erklärung dafür, dass es die richtige Wahrscheinlichkeitsdichte erzeugt, wenn es verwendet wird, um eine andere willkürliche Wellenfunktion zu überspannen Ψ ( X ) . Das Problem ist natürlich, dass die oben angegebene Wellenfunktion zu einer nicht integrierbaren Funktion quadriert. Soweit ich weiß, ist dieses Problem ungelöst. Meine Frage ist, ob jemand den Status quo dieses Problems kennt. Es tut mir leid, wenn diese Frage doppelt vorhanden ist, ich konnte sie nicht unter den beantworteten Fragen finden.

Es sollte betont werden, dass der manipulierte Hilbert-Raum-Formalismus die Bedeutung des Integrals des Quadrats der Dirac-Verteilung nicht erklärt.

Antworten (3)

Mathematisch gesprochen müssen Ihre Wellenfunktionen in sein, da Sie möchten, dass Ihre Wellenfunktionen quadratisch integrierbar sind L 2 oder irgendein Unterraum davon. Allerdings werden Sie in diesem Raum keine Funktion finden, die einen Träger auf einer zählbaren Menge von Punkten hat, da das Lebesgue-Integral keine zählbaren Mengen sehen kann (Maß 0), daher kann es keine Funktion (dh keine Wellenfunktion) mit Träger geben in einem einzigen Punkt (übrigens ist die Delta-Funktion aus diesem Grund in gewisser Weise keine "Funktion").

Dies sagt uns, dass eine Wellenfunktion für ein vollständig lokalisiertes Teilchen nicht in der üblichen Anordnung quadratischer Lebesgue-integrierbarer Funktionen definiert werden kann, was nicht allzu tragisch ist, weil wir es sowieso nicht für physikalisch sinnvoll halten.

@Martin-Warum ist das so? Nach Positionsmessung auf einem System erstellen wir diese δ - Funktionszustände ... meinst du, sie sind unphysikalisch?
@Roopam Nein, wir erstellen diese Delta-Funktionen nicht, weil wir die Position nicht genau messen können. Wir können die Position in Wirklichkeit nicht perfekt messen, denn welches Instrument auch immer wir bauen können, es wird immer die Position in einem diskreten Satz messen - und dann wird die resultierende projizierte Wellenfunktion ein Wellenpaket sein, das schön quadratisch integrierbar ist. In diesem Sinne sind Deltafunktionen also unphysikalisch. Die Delta-Funktion ist nur eine physikalisch-mathematische Annäherung, die uns das Leben erleichtert. Die Handhabung geeigneter Messungen von Positionen würde andernfalls eine geeignete Maßtheorie erfordern.
@Martin- Aber zumindest verbietet das Unsicherheitsprinzip nicht die genaue Positionsmessung, wenn wir uns gleichzeitig nicht um seinen genauen Impuls kümmern. Glauben Sie also nicht, dass es prinzipiell kein Problem mit einer präzisen Positionsmessung gibt?
Aus meiner Sicht gibt es keinen anderen mathematischen Grund, uns genaue Positionsmessungen zu verbieten, als dass der resultierende Zustand nicht in unserem Zustandsraum liegt - aber das ist nur ein Definitionsproblem. Wir könnten doch mit manipulierten Hilbert-Räumen arbeiten. Aber es gibt einen genauen PHYSIKALISCHEN Grund: Wir messen, indem wir eine Zahl oder so etwas messen, aber wir werden vielleicht nie unendlich viele Ziffern messen - eine gewisse Digitalisierung wird es immer geben.
Alles in allem gibt es also zwei Dinge: a) Unser Formalismus sagt per Definition, dass alle physikalischen Objekte durch Wellenfunktionen repräsentiert werden, die quadratisch integrierbare Objekte sind, um eine Wahrscheinlichkeitsinterpretation zu haben. b) Wir stoßen auf Dinge, die nicht quadratintegrierbar sind - daher müssen sie per Definition unphysikalisch sein, damit wir entweder einen physikalischen Grund finden können, warum sie unphysikalisch sind, oder wir müssen unseren Formalismus erweitern. Da ich eine rein physikalische Begründung gegeben habe (Messen mit beliebiger, aber endlicher Genauigkeit ist möglich, unendliche Genauigkeit ist nicht möglich), ist das in Ordnung.
@Martin-Ja. Diese sind unphysikalisch. Aber Dirac sagt in seinem Buch Prinzipien der Quantenmechanik, dass Physiker den Hilbert-Raum erweitern, um auch Vektoren mit unendlicher Norm einzubeziehen. Es ist also nicht mehr nur L 2 . Jene in L 2 sind körperlich, aber außerhalb L 2 sind nicht körperlich. Da müssen wir mit einer überabzählbar unendlichen Basis wie arbeiten { | X } , die nicht drin sind L 2 , es ist besser, den Hilbert-Raum zu vergrößern und nicht darauf beschränkt zu bleiben L 2 .
@Roopam Ja, aber das ist nebensächlich. "Unphysikalische Zustände" können in der Natur nicht existieren (daher ihr Name), also müssen wir uns keine Sorgen um ihre Normalisierung machen - darum ging es in der ursprünglichen Frage. Natürlich können wir sie im nicht normalisieren L 2 -weg, weil sie es nicht sind L 2 . Außerdem sind Sie nicht verpflichtet, mit "Basen" wie zu arbeiten { | X } - es ist einfach sehr bequem, dies zu tun.
Martins Punkte sollten gut genommen werden. Keine physikalisch realisierbare Messung kann zu einer perfekten reellen Zahl führen. Tatsächlich und allgemeiner hat keine physikalische Manifestation irgendeines Zustands einen einzigen reellen Eigenwert. Ein solcher Zustand wäre ein Null-Entropie-Zustand und dies ist durch den dritten Hauptsatz der Thermodynamik verboten. Jeder physikalisch realisierbare Zustand hat ein quadratintegrierbares Spektrum von Eigenwerten, ein Spektrum von Nicht-Null-Maß. Eine Delta-Funktion hat ein Nullmaß in x und ist daher nicht physikalisch.

Ich kann Martins Antwort nicht verstehen, obwohl ich denke, dass es eine ausgezeichnete physikalische Antwort auf das OP gibt.

Die meisten Leute vergessen, dass das Delta eines Diracs durch sehr viele Funktionen angenähert werden kann, da einige Parameter (z. σ ) tendiert gegen Null. Eine solche Klasse von Funktionen ist natürlich F ( X ) = ( π / σ ) e X P ( X ² / σ ² ) . Nehmen wir dies als Annäherung an die Wahrscheinlichkeitsverteilung ( nicht die Wellenfunktion) eines Quantenzustands, der ein Teilchen darstellt, das auf den Koordinatenursprung beschränkt ist. Diese Funktion ist glatt, integrierbar, auf 1 normiert.

Die zugehörige Wellenfunktion, ϕ ( X ) = ( π / σ ) 1 / 2 e X P ( X ² / ( 2 σ ² ) ) ebenfalls glatt und über dasselbe Intervall integrierbar ist, und der Wert dieses Integrals 0 als σ 0 .

Aber wir kümmern uns nicht wirklich um die Integrierbarkeit der Wellenfunktion an sich , sondern nur darum, dass sie sinnvolle Ergebnisse liefert, wenn wir Übergangsamplituden berechnen. Und für eine beliebige Wellenfunktion ψ ( X ) die Übergangsamplitude ist immer proportional zu

ψ ( X ) ϕ ( X ) D X

was sich aus obigem in der Grenze leicht berechnen lässt σ 0 : es gibt immer nach 0 , es sei denn ψ ( X ) = ϕ ( X ) , in diesem Fall gibt es nach 1 . Macht das Sinn? Ja, das tut es: wann immer ψ nicht die Wellenfunktion eines am Ursprung eingeschlossenen Teilchens ist ( dh wenn es ein an anderer Stelle eingeschlossenes Teilchen darstellt, oder wenn es ein überhaupt nicht eingeschlossenes Teilchen darstellt), sind die beiden Wellenfunktionen orthogonal, weil sie völlig unterschiedliche physikalische Zustände darstellen: das Integral Oben (wenn quadriert) ist die Wahrscheinlichkeit, dass ein beliebiger Quantenzustand genau am Ursprung gefunden wird, der natürlich sowohl für glatte Wahrscheinlichkeitsverteilungen als auch für Teilchen, die an einem Punkt eingeschlossen sind, der nicht der Ursprung ist, Null ist.

Daher macht es durchaus Sinn, dass das obige Integral verschwindet, es sei denn ψ = ϕ wenn es Gewissheit geben muss ( = 1 ).

Zusammenfassend lässt sich sagen, dass die Wellenfunktion eines Zustands, der ein am Ursprung eingeschlossenes Teilchen darstellt, existiert, glatt und integrierbar ist, solange σ > 0 , aber darüber machen wir uns (zumindest aus physikalischer Sicht) keine Gedanken, weil die Wellenfunktion an sich keine Observable ist, denn alles, was uns interessiert, ist, dass Übergangsamplituden existieren, und diese existieren und machen sogar in der physikalisch Sinn Grenze σ 0 .

Die Methode, ein Dirac-Delta durch seine Approximanten zu ersetzen, führt oft zu recht vernünftigen Antworten.

1, Hier gibt es ein kleines Problem, dass Sie eine Wellenfunktion zu nehmen scheinen ϕ σ so dass lim σ 0 ϕ σ ϕ σ = δ , anstatt dass die Wellenfunktion selbst die entstehende ist δ . 2. Es gibt hier ein großes Problem, dass nicht klar ist, wo Sie die Grenzen nehmen und wo die Grenzen liegen. Innerhalb des Hilbert-Raums von L 2 Funktionen, die Sequenz ϕ σ konvergiert gegen 0; man muss sorgfältig angeben, in welchem ​​Raum die Grenze genommen werden muss, um a zu liefern δ .
@ACuriousMind Ich stimme Punkt eins zu: Ich nehme an, dass die Wahrscheinlichkeitsverteilung ein Dirac-Delta ist, um den Zustand eines am Ursprung begrenzten Teilchens darzustellen. Physikalisch kann ich nicht verstehen, was für ein Zustand einer mit einer Wellenfunktion in Form eines Dirac-Deltas ist, oder? Bei Punkt 2 bin ich anderer Meinung: Die Grenze der Funktionsfolge ist das, was Sie darin angeben L 2 in der Tat, aber es ist ein Diracsches Delta, wenn wir den Raum stetiger Funktionale (also Verteilungen) verwenden L 1 Funktionen statt L 2 . Schauen Sie, das ist eine ganz normale Verteilungstheorie von Laurent Schwartz.
@ACuriousMind Hast du schon einmal von den sogenannten Delta-Approximationen gehört ? Schlagen Sie sie hier nach, en.wikipedia.org/wiki/…
Ich weiß das alles, aber es ist mir nicht klar, warum die schwache Konvergenz im Sinne von Verteilungen als der physikalisch sinnvolle Begriff der Konvergenz für Zustände angesehen werden sollte . Die Zustände sind Elemente (oder Strahlen) des Hilbert-Raums, daher erscheint es mir natürlich, dass der physikalisch relevante Begriff der Konvergenz der der Konvergenz in der Hilbert-Raum-Norm ist.
@ACuriousMind Du meinst, abgesehen vom physischen Sinn, der das sagt ϕ ( X ) beschreibt oben ein sehr eng begrenztes Teilchen? Nun, dann können Sie Wahrscheinlichkeitsverteilungen als Funktionale sehen, die auf den Raum dynamischer Größen (Energie, Ort, linearer und Drehimpuls, Ladungsverteilung und Quadrupolmoment, ...) wirken, um Skalare, dh Messungen, zu ergeben .

Die meisten Wissenschaftler sind sich einig, dass QM immer noch einige Interpretationsprobleme hat, daher ist es schwierig, eindeutige Aussagen zu treffen. Meiner Meinung nach sind punktartige reine Quantenzustände (dh Hilbert-Raum-"Strahlen") nicht physikalisch messbar oder sogar physikalisch realisierbar. Sie sind Idealisierungen der Nullentropie und somit nicht durch die Nernst-Aussage des dritten Hauptsatzes der Thermodynamik realisierbar. Keine Eigenschaft eines physikalischen Objekts kann Existenz in einem Raumzeitintervall der Ausdehnung null manifestieren, z. B. kann kein Objekt eine reelle Position manifestieren, nicht einmal sein CG. Das bedeutet, dass jeder realisierbare Quantenzustand ein Mischzustand aus mehreren gleichzeitig existierenden inkohärenten reinen Zuständen ist. (Sie sind inkohärente Überlagerungen, sonst wären sie als kohärente Summe reiner Zustände ausdrückbar, dh eine einzelne Komponente, null Entropie, reiner Zustand.) Dies jedoch, lässt Zweifel an der Wahrscheinlichkeitsinterpretation von QM aufkommen. Die geeignete vertikale Metrik, die auf das Betragsquadrat einer realisierbaren Quantenmischzustandsverteilung angewendet werden sollte, muss eine physikalische oder ontische Metrik sein, keine Wahrscheinlichkeit. OP, reelle (punktartige) Eigenwerte sind also nicht physikalisch realisierbar, alle manifesten physikalischen Eigenschaftswerte sind Verteilungen, keine reellen Zahlen.